Computing the volume of a simplex-like object with constraintspigeonhole principle Number of integer combinations x_1 < … < x_n ? Simplest form for sum of Binomial ExpressionsModules over quantum complete intersectionsAn inequality related to Lagrange's identity and $L_p$ normA nested integral sequencePartition of 4-tuplesEvaluating a Fermi gas problem for a SO(2N+1) matrix integralTuples with same coordinate sum4-tuples with close sums

Computing the volume of a simplex-like object with constraints


pigeonhole principle Number of integer combinations x_1 < … < x_n ? Simplest form for sum of Binomial ExpressionsModules over quantum complete intersectionsAn inequality related to Lagrange's identity and $L_p$ normA nested integral sequencePartition of 4-tuplesEvaluating a Fermi gas problem for a SO(2N+1) matrix integralTuples with same coordinate sum4-tuples with close sums













6












$begingroup$


For any $n geq 2$, let
$$D_n [r , (a_1, b_1 ) , ldots , (a_n, b_n) ] =
(x_1 , ldots , x_n ) in mathbb R^n mid
sum_i x_i = r mbox and b_i geq x_i geq a_i , forall i ,$$

where $r geq b_i geq a_i geq 0$ for all $i$, and all are real numbers.




Question: What is the 'volume' of $D_n [r , (a_1, b_1 ) , ldots , (a_n, b_n) ]$?




So for example for $n=2$, this set is either empty or it is some short line, and the purpose would be to calculate the length of that line (in terms of the parameters $r, a_i, b_i$). This is easy, I've done that already. Also the case $n=3$ would in principle still be doable to do by hand: it would be either zero (in case the set is empty) or part of a plane in $mathbb R^3$, the area of which we desire to compute.



Now to come up with a formula for the case $n=3$ (and higher) in a smarter way, my idea was to reason inductively from the case $n=2$, so basically reducing the three dimensional case to the two dimensional one, etc. I obviously tried to use integrals.



The problem I run into, is that in order to compute the volume we want with integrals, we have to view this set as (a subset of) an $n-1$-dimensional space. (Indeed, the integral of the constant function $1$ over the region $D_n [r , (a_1, b_1 ) , ldots , (a_n, b_n) ]$ seen as part of $mathbb R^n$ (rather than $mathbb R^n-1$), is equal to $0$. That's not what we want.) But to do that, it seems to me that we would need a concrete isometric embedding of $D_n [r , (a_1, b_1 ) , ldots , (a_n, b_n) ]$ into $mathbb R^n-1$, and I can't really find a nice one.



Do you have an idea about how to approach this problem best?



(This question was previously posted on Math.StackExchange, see https://math.stackexchange.com/questions/3135606/computing-hyper-area-of-a-contrained-simplex.)










share|cite|improve this question









New contributor




Oscar W. is a new contributor to this site. Take care in asking for clarification, commenting, and answering.
Check out our Code of Conduct.







$endgroup$
















    6












    $begingroup$


    For any $n geq 2$, let
    $$D_n [r , (a_1, b_1 ) , ldots , (a_n, b_n) ] =
    (x_1 , ldots , x_n ) in mathbb R^n mid
    sum_i x_i = r mbox and b_i geq x_i geq a_i , forall i ,$$

    where $r geq b_i geq a_i geq 0$ for all $i$, and all are real numbers.




    Question: What is the 'volume' of $D_n [r , (a_1, b_1 ) , ldots , (a_n, b_n) ]$?




    So for example for $n=2$, this set is either empty or it is some short line, and the purpose would be to calculate the length of that line (in terms of the parameters $r, a_i, b_i$). This is easy, I've done that already. Also the case $n=3$ would in principle still be doable to do by hand: it would be either zero (in case the set is empty) or part of a plane in $mathbb R^3$, the area of which we desire to compute.



    Now to come up with a formula for the case $n=3$ (and higher) in a smarter way, my idea was to reason inductively from the case $n=2$, so basically reducing the three dimensional case to the two dimensional one, etc. I obviously tried to use integrals.



    The problem I run into, is that in order to compute the volume we want with integrals, we have to view this set as (a subset of) an $n-1$-dimensional space. (Indeed, the integral of the constant function $1$ over the region $D_n [r , (a_1, b_1 ) , ldots , (a_n, b_n) ]$ seen as part of $mathbb R^n$ (rather than $mathbb R^n-1$), is equal to $0$. That's not what we want.) But to do that, it seems to me that we would need a concrete isometric embedding of $D_n [r , (a_1, b_1 ) , ldots , (a_n, b_n) ]$ into $mathbb R^n-1$, and I can't really find a nice one.



    Do you have an idea about how to approach this problem best?



    (This question was previously posted on Math.StackExchange, see https://math.stackexchange.com/questions/3135606/computing-hyper-area-of-a-contrained-simplex.)










    share|cite|improve this question









    New contributor




    Oscar W. is a new contributor to this site. Take care in asking for clarification, commenting, and answering.
    Check out our Code of Conduct.







    $endgroup$














      6












      6








      6


      1



      $begingroup$


      For any $n geq 2$, let
      $$D_n [r , (a_1, b_1 ) , ldots , (a_n, b_n) ] =
      (x_1 , ldots , x_n ) in mathbb R^n mid
      sum_i x_i = r mbox and b_i geq x_i geq a_i , forall i ,$$

      where $r geq b_i geq a_i geq 0$ for all $i$, and all are real numbers.




      Question: What is the 'volume' of $D_n [r , (a_1, b_1 ) , ldots , (a_n, b_n) ]$?




      So for example for $n=2$, this set is either empty or it is some short line, and the purpose would be to calculate the length of that line (in terms of the parameters $r, a_i, b_i$). This is easy, I've done that already. Also the case $n=3$ would in principle still be doable to do by hand: it would be either zero (in case the set is empty) or part of a plane in $mathbb R^3$, the area of which we desire to compute.



      Now to come up with a formula for the case $n=3$ (and higher) in a smarter way, my idea was to reason inductively from the case $n=2$, so basically reducing the three dimensional case to the two dimensional one, etc. I obviously tried to use integrals.



      The problem I run into, is that in order to compute the volume we want with integrals, we have to view this set as (a subset of) an $n-1$-dimensional space. (Indeed, the integral of the constant function $1$ over the region $D_n [r , (a_1, b_1 ) , ldots , (a_n, b_n) ]$ seen as part of $mathbb R^n$ (rather than $mathbb R^n-1$), is equal to $0$. That's not what we want.) But to do that, it seems to me that we would need a concrete isometric embedding of $D_n [r , (a_1, b_1 ) , ldots , (a_n, b_n) ]$ into $mathbb R^n-1$, and I can't really find a nice one.



      Do you have an idea about how to approach this problem best?



      (This question was previously posted on Math.StackExchange, see https://math.stackexchange.com/questions/3135606/computing-hyper-area-of-a-contrained-simplex.)










      share|cite|improve this question









      New contributor




      Oscar W. is a new contributor to this site. Take care in asking for clarification, commenting, and answering.
      Check out our Code of Conduct.







      $endgroup$




      For any $n geq 2$, let
      $$D_n [r , (a_1, b_1 ) , ldots , (a_n, b_n) ] =
      (x_1 , ldots , x_n ) in mathbb R^n mid
      sum_i x_i = r mbox and b_i geq x_i geq a_i , forall i ,$$

      where $r geq b_i geq a_i geq 0$ for all $i$, and all are real numbers.




      Question: What is the 'volume' of $D_n [r , (a_1, b_1 ) , ldots , (a_n, b_n) ]$?




      So for example for $n=2$, this set is either empty or it is some short line, and the purpose would be to calculate the length of that line (in terms of the parameters $r, a_i, b_i$). This is easy, I've done that already. Also the case $n=3$ would in principle still be doable to do by hand: it would be either zero (in case the set is empty) or part of a plane in $mathbb R^3$, the area of which we desire to compute.



      Now to come up with a formula for the case $n=3$ (and higher) in a smarter way, my idea was to reason inductively from the case $n=2$, so basically reducing the three dimensional case to the two dimensional one, etc. I obviously tried to use integrals.



      The problem I run into, is that in order to compute the volume we want with integrals, we have to view this set as (a subset of) an $n-1$-dimensional space. (Indeed, the integral of the constant function $1$ over the region $D_n [r , (a_1, b_1 ) , ldots , (a_n, b_n) ]$ seen as part of $mathbb R^n$ (rather than $mathbb R^n-1$), is equal to $0$. That's not what we want.) But to do that, it seems to me that we would need a concrete isometric embedding of $D_n [r , (a_1, b_1 ) , ldots , (a_n, b_n) ]$ into $mathbb R^n-1$, and I can't really find a nice one.



      Do you have an idea about how to approach this problem best?



      (This question was previously posted on Math.StackExchange, see https://math.stackexchange.com/questions/3135606/computing-hyper-area-of-a-contrained-simplex.)







      co.combinatorics real-analysis integration






      share|cite|improve this question









      New contributor




      Oscar W. is a new contributor to this site. Take care in asking for clarification, commenting, and answering.
      Check out our Code of Conduct.











      share|cite|improve this question









      New contributor




      Oscar W. is a new contributor to this site. Take care in asking for clarification, commenting, and answering.
      Check out our Code of Conduct.









      share|cite|improve this question




      share|cite|improve this question








      edited 11 hours ago









      Iosif Pinelis

      19.5k22259




      19.5k22259






      New contributor




      Oscar W. is a new contributor to this site. Take care in asking for clarification, commenting, and answering.
      Check out our Code of Conduct.









      asked 12 hours ago









      Oscar W.Oscar W.

      311




      311




      New contributor




      Oscar W. is a new contributor to this site. Take care in asking for clarification, commenting, and answering.
      Check out our Code of Conduct.





      New contributor





      Oscar W. is a new contributor to this site. Take care in asking for clarification, commenting, and answering.
      Check out our Code of Conduct.






      Oscar W. is a new contributor to this site. Take care in asking for clarification, commenting, and answering.
      Check out our Code of Conduct.




















          3 Answers
          3






          active

          oldest

          votes


















          5












          $begingroup$

          Computing volumes of polytopes in general is NP-hard. However, your polytope is special - it is a slice of a hypercube by a hyperplane, and this is tractable, see Theorem 1 in:



          Marichal, Jean-Luc; Mossinghoff, Michael J., Slices, slabs, and sections of the unit hypercube, Online J. Anal. Comb. 3, Article 1, 11 p. (2008). ZBL1189.52011.






          share|cite|improve this answer









          $endgroup$




















            2












            $begingroup$

            The $(n-1)$-volume of your polytope (in $mathbb R^n$) equals the $(n-1)$-volume of the polytope
            beginmultline
            P:=(x_1,dots,x_n-1)inmathbb R^n-1colon \
            a_ile x_ile b_i forall i=1,dots,n-1,\
            a_nle r-sum_1^n-1x_ile b_n
            endmultline

            (in $mathbb R^n-1$) divided by $1/sqrt n$, which latter is the cosine of the angle between the unit vectors $(1/sqrt n,dots,1/sqrt n)$ and $(0,dots,0,1)$ in $mathbb R^n$ -- because $P$ is the image of your polytope under the orthogonal projection of $mathbb R^n$ onto $mathbb R^n-1$ given by $(x_1,dots,x_n)mapsto(x_1,dots,x_n-1)$. The unit vectors $(1/sqrt n,dots,1/sqrt n)$ and $(0,dots,0,1)$ are normal vectors to, respectively, the hyperplane containing your polytope and the hyperplane $(x_1,dots,x_n)inmathbb R^ncolon x_n=0$; the latter hyperplane is identified with $mathbb R^n-1$.



            A formula for the volume of a polytope was given by Lawrence.






            share|cite|improve this answer











            $endgroup$




















              2












              $begingroup$

              We may assume without loss of generality that $a_i=0$. If
              $r$ and each $b_i$ are positive integers, then consider
              $$ f(x) = fracleft( 1-x^tb_1+1right)cdots
              left( 1-x^tb_n+1right)(1-x)^n. $$

              The coefficient of $x^tr$ is a polynomial function of $t$,
              and the volume $V$ will be its leading coefficient. If I didn't
              make a computational error, then
              $$ V=frac1(n-1)!sum_substackSsubseteq
              1,dots,n\ sum_iin Sb_i<r (-1)^S
              left(r-sum_iin Sb_iright)^n-1. $$

              If this isn't correct, then something close to it will be.






              share|cite|improve this answer









              $endgroup$












                Your Answer





                StackExchange.ifUsing("editor", function ()
                return StackExchange.using("mathjaxEditing", function ()
                StackExchange.MarkdownEditor.creationCallbacks.add(function (editor, postfix)
                StackExchange.mathjaxEditing.prepareWmdForMathJax(editor, postfix, [["$", "$"], ["\\(","\\)"]]);
                );
                );
                , "mathjax-editing");

                StackExchange.ready(function()
                var channelOptions =
                tags: "".split(" "),
                id: "504"
                ;
                initTagRenderer("".split(" "), "".split(" "), channelOptions);

                StackExchange.using("externalEditor", function()
                // Have to fire editor after snippets, if snippets enabled
                if (StackExchange.settings.snippets.snippetsEnabled)
                StackExchange.using("snippets", function()
                createEditor();
                );

                else
                createEditor();

                );

                function createEditor()
                StackExchange.prepareEditor(
                heartbeatType: 'answer',
                autoActivateHeartbeat: false,
                convertImagesToLinks: true,
                noModals: true,
                showLowRepImageUploadWarning: true,
                reputationToPostImages: 10,
                bindNavPrevention: true,
                postfix: "",
                imageUploader:
                brandingHtml: "Powered by u003ca class="icon-imgur-white" href="https://imgur.com/"u003eu003c/au003e",
                contentPolicyHtml: "User contributions licensed under u003ca href="https://creativecommons.org/licenses/by-sa/3.0/"u003ecc by-sa 3.0 with attribution requiredu003c/au003e u003ca href="https://stackoverflow.com/legal/content-policy"u003e(content policy)u003c/au003e",
                allowUrls: true
                ,
                noCode: true, onDemand: true,
                discardSelector: ".discard-answer"
                ,immediatelyShowMarkdownHelp:true
                );



                );






                Oscar W. is a new contributor. Be nice, and check out our Code of Conduct.









                draft saved

                draft discarded


















                StackExchange.ready(
                function ()
                StackExchange.openid.initPostLogin('.new-post-login', 'https%3a%2f%2fmathoverflow.net%2fquestions%2f324878%2fcomputing-the-volume-of-a-simplex-like-object-with-constraints%23new-answer', 'question_page');

                );

                Post as a guest















                Required, but never shown

























                3 Answers
                3






                active

                oldest

                votes








                3 Answers
                3






                active

                oldest

                votes









                active

                oldest

                votes






                active

                oldest

                votes









                5












                $begingroup$

                Computing volumes of polytopes in general is NP-hard. However, your polytope is special - it is a slice of a hypercube by a hyperplane, and this is tractable, see Theorem 1 in:



                Marichal, Jean-Luc; Mossinghoff, Michael J., Slices, slabs, and sections of the unit hypercube, Online J. Anal. Comb. 3, Article 1, 11 p. (2008). ZBL1189.52011.






                share|cite|improve this answer









                $endgroup$

















                  5












                  $begingroup$

                  Computing volumes of polytopes in general is NP-hard. However, your polytope is special - it is a slice of a hypercube by a hyperplane, and this is tractable, see Theorem 1 in:



                  Marichal, Jean-Luc; Mossinghoff, Michael J., Slices, slabs, and sections of the unit hypercube, Online J. Anal. Comb. 3, Article 1, 11 p. (2008). ZBL1189.52011.






                  share|cite|improve this answer









                  $endgroup$















                    5












                    5








                    5





                    $begingroup$

                    Computing volumes of polytopes in general is NP-hard. However, your polytope is special - it is a slice of a hypercube by a hyperplane, and this is tractable, see Theorem 1 in:



                    Marichal, Jean-Luc; Mossinghoff, Michael J., Slices, slabs, and sections of the unit hypercube, Online J. Anal. Comb. 3, Article 1, 11 p. (2008). ZBL1189.52011.






                    share|cite|improve this answer









                    $endgroup$



                    Computing volumes of polytopes in general is NP-hard. However, your polytope is special - it is a slice of a hypercube by a hyperplane, and this is tractable, see Theorem 1 in:



                    Marichal, Jean-Luc; Mossinghoff, Michael J., Slices, slabs, and sections of the unit hypercube, Online J. Anal. Comb. 3, Article 1, 11 p. (2008). ZBL1189.52011.







                    share|cite|improve this answer












                    share|cite|improve this answer



                    share|cite|improve this answer










                    answered 10 hours ago









                    Igor RivinIgor Rivin

                    79.5k8113309




                    79.5k8113309





















                        2












                        $begingroup$

                        The $(n-1)$-volume of your polytope (in $mathbb R^n$) equals the $(n-1)$-volume of the polytope
                        beginmultline
                        P:=(x_1,dots,x_n-1)inmathbb R^n-1colon \
                        a_ile x_ile b_i forall i=1,dots,n-1,\
                        a_nle r-sum_1^n-1x_ile b_n
                        endmultline

                        (in $mathbb R^n-1$) divided by $1/sqrt n$, which latter is the cosine of the angle between the unit vectors $(1/sqrt n,dots,1/sqrt n)$ and $(0,dots,0,1)$ in $mathbb R^n$ -- because $P$ is the image of your polytope under the orthogonal projection of $mathbb R^n$ onto $mathbb R^n-1$ given by $(x_1,dots,x_n)mapsto(x_1,dots,x_n-1)$. The unit vectors $(1/sqrt n,dots,1/sqrt n)$ and $(0,dots,0,1)$ are normal vectors to, respectively, the hyperplane containing your polytope and the hyperplane $(x_1,dots,x_n)inmathbb R^ncolon x_n=0$; the latter hyperplane is identified with $mathbb R^n-1$.



                        A formula for the volume of a polytope was given by Lawrence.






                        share|cite|improve this answer











                        $endgroup$

















                          2












                          $begingroup$

                          The $(n-1)$-volume of your polytope (in $mathbb R^n$) equals the $(n-1)$-volume of the polytope
                          beginmultline
                          P:=(x_1,dots,x_n-1)inmathbb R^n-1colon \
                          a_ile x_ile b_i forall i=1,dots,n-1,\
                          a_nle r-sum_1^n-1x_ile b_n
                          endmultline

                          (in $mathbb R^n-1$) divided by $1/sqrt n$, which latter is the cosine of the angle between the unit vectors $(1/sqrt n,dots,1/sqrt n)$ and $(0,dots,0,1)$ in $mathbb R^n$ -- because $P$ is the image of your polytope under the orthogonal projection of $mathbb R^n$ onto $mathbb R^n-1$ given by $(x_1,dots,x_n)mapsto(x_1,dots,x_n-1)$. The unit vectors $(1/sqrt n,dots,1/sqrt n)$ and $(0,dots,0,1)$ are normal vectors to, respectively, the hyperplane containing your polytope and the hyperplane $(x_1,dots,x_n)inmathbb R^ncolon x_n=0$; the latter hyperplane is identified with $mathbb R^n-1$.



                          A formula for the volume of a polytope was given by Lawrence.






                          share|cite|improve this answer











                          $endgroup$















                            2












                            2








                            2





                            $begingroup$

                            The $(n-1)$-volume of your polytope (in $mathbb R^n$) equals the $(n-1)$-volume of the polytope
                            beginmultline
                            P:=(x_1,dots,x_n-1)inmathbb R^n-1colon \
                            a_ile x_ile b_i forall i=1,dots,n-1,\
                            a_nle r-sum_1^n-1x_ile b_n
                            endmultline

                            (in $mathbb R^n-1$) divided by $1/sqrt n$, which latter is the cosine of the angle between the unit vectors $(1/sqrt n,dots,1/sqrt n)$ and $(0,dots,0,1)$ in $mathbb R^n$ -- because $P$ is the image of your polytope under the orthogonal projection of $mathbb R^n$ onto $mathbb R^n-1$ given by $(x_1,dots,x_n)mapsto(x_1,dots,x_n-1)$. The unit vectors $(1/sqrt n,dots,1/sqrt n)$ and $(0,dots,0,1)$ are normal vectors to, respectively, the hyperplane containing your polytope and the hyperplane $(x_1,dots,x_n)inmathbb R^ncolon x_n=0$; the latter hyperplane is identified with $mathbb R^n-1$.



                            A formula for the volume of a polytope was given by Lawrence.






                            share|cite|improve this answer











                            $endgroup$



                            The $(n-1)$-volume of your polytope (in $mathbb R^n$) equals the $(n-1)$-volume of the polytope
                            beginmultline
                            P:=(x_1,dots,x_n-1)inmathbb R^n-1colon \
                            a_ile x_ile b_i forall i=1,dots,n-1,\
                            a_nle r-sum_1^n-1x_ile b_n
                            endmultline

                            (in $mathbb R^n-1$) divided by $1/sqrt n$, which latter is the cosine of the angle between the unit vectors $(1/sqrt n,dots,1/sqrt n)$ and $(0,dots,0,1)$ in $mathbb R^n$ -- because $P$ is the image of your polytope under the orthogonal projection of $mathbb R^n$ onto $mathbb R^n-1$ given by $(x_1,dots,x_n)mapsto(x_1,dots,x_n-1)$. The unit vectors $(1/sqrt n,dots,1/sqrt n)$ and $(0,dots,0,1)$ are normal vectors to, respectively, the hyperplane containing your polytope and the hyperplane $(x_1,dots,x_n)inmathbb R^ncolon x_n=0$; the latter hyperplane is identified with $mathbb R^n-1$.



                            A formula for the volume of a polytope was given by Lawrence.







                            share|cite|improve this answer














                            share|cite|improve this answer



                            share|cite|improve this answer








                            edited 10 hours ago

























                            answered 10 hours ago









                            Iosif PinelisIosif Pinelis

                            19.5k22259




                            19.5k22259





















                                2












                                $begingroup$

                                We may assume without loss of generality that $a_i=0$. If
                                $r$ and each $b_i$ are positive integers, then consider
                                $$ f(x) = fracleft( 1-x^tb_1+1right)cdots
                                left( 1-x^tb_n+1right)(1-x)^n. $$

                                The coefficient of $x^tr$ is a polynomial function of $t$,
                                and the volume $V$ will be its leading coefficient. If I didn't
                                make a computational error, then
                                $$ V=frac1(n-1)!sum_substackSsubseteq
                                1,dots,n\ sum_iin Sb_i<r (-1)^S
                                left(r-sum_iin Sb_iright)^n-1. $$

                                If this isn't correct, then something close to it will be.






                                share|cite|improve this answer









                                $endgroup$

















                                  2












                                  $begingroup$

                                  We may assume without loss of generality that $a_i=0$. If
                                  $r$ and each $b_i$ are positive integers, then consider
                                  $$ f(x) = fracleft( 1-x^tb_1+1right)cdots
                                  left( 1-x^tb_n+1right)(1-x)^n. $$

                                  The coefficient of $x^tr$ is a polynomial function of $t$,
                                  and the volume $V$ will be its leading coefficient. If I didn't
                                  make a computational error, then
                                  $$ V=frac1(n-1)!sum_substackSsubseteq
                                  1,dots,n\ sum_iin Sb_i<r (-1)^S
                                  left(r-sum_iin Sb_iright)^n-1. $$

                                  If this isn't correct, then something close to it will be.






                                  share|cite|improve this answer









                                  $endgroup$















                                    2












                                    2








                                    2





                                    $begingroup$

                                    We may assume without loss of generality that $a_i=0$. If
                                    $r$ and each $b_i$ are positive integers, then consider
                                    $$ f(x) = fracleft( 1-x^tb_1+1right)cdots
                                    left( 1-x^tb_n+1right)(1-x)^n. $$

                                    The coefficient of $x^tr$ is a polynomial function of $t$,
                                    and the volume $V$ will be its leading coefficient. If I didn't
                                    make a computational error, then
                                    $$ V=frac1(n-1)!sum_substackSsubseteq
                                    1,dots,n\ sum_iin Sb_i<r (-1)^S
                                    left(r-sum_iin Sb_iright)^n-1. $$

                                    If this isn't correct, then something close to it will be.






                                    share|cite|improve this answer









                                    $endgroup$



                                    We may assume without loss of generality that $a_i=0$. If
                                    $r$ and each $b_i$ are positive integers, then consider
                                    $$ f(x) = fracleft( 1-x^tb_1+1right)cdots
                                    left( 1-x^tb_n+1right)(1-x)^n. $$

                                    The coefficient of $x^tr$ is a polynomial function of $t$,
                                    and the volume $V$ will be its leading coefficient. If I didn't
                                    make a computational error, then
                                    $$ V=frac1(n-1)!sum_substackSsubseteq
                                    1,dots,n\ sum_iin Sb_i<r (-1)^S
                                    left(r-sum_iin Sb_iright)^n-1. $$

                                    If this isn't correct, then something close to it will be.







                                    share|cite|improve this answer












                                    share|cite|improve this answer



                                    share|cite|improve this answer










                                    answered 8 hours ago









                                    Richard StanleyRichard Stanley

                                    28.9k9115189




                                    28.9k9115189




















                                        Oscar W. is a new contributor. Be nice, and check out our Code of Conduct.









                                        draft saved

                                        draft discarded


















                                        Oscar W. is a new contributor. Be nice, and check out our Code of Conduct.












                                        Oscar W. is a new contributor. Be nice, and check out our Code of Conduct.











                                        Oscar W. is a new contributor. Be nice, and check out our Code of Conduct.














                                        Thanks for contributing an answer to MathOverflow!


                                        • Please be sure to answer the question. Provide details and share your research!

                                        But avoid


                                        • Asking for help, clarification, or responding to other answers.

                                        • Making statements based on opinion; back them up with references or personal experience.

                                        Use MathJax to format equations. MathJax reference.


                                        To learn more, see our tips on writing great answers.




                                        draft saved


                                        draft discarded














                                        StackExchange.ready(
                                        function ()
                                        StackExchange.openid.initPostLogin('.new-post-login', 'https%3a%2f%2fmathoverflow.net%2fquestions%2f324878%2fcomputing-the-volume-of-a-simplex-like-object-with-constraints%23new-answer', 'question_page');

                                        );

                                        Post as a guest















                                        Required, but never shown





















































                                        Required, but never shown














                                        Required, but never shown












                                        Required, but never shown







                                        Required, but never shown

































                                        Required, but never shown














                                        Required, but never shown












                                        Required, but never shown







                                        Required, but never shown







                                        Popular posts from this blog

                                        Magento 2 - Add success message with knockout Planned maintenance scheduled April 23, 2019 at 23:30 UTC (7:30pm US/Eastern) Announcing the arrival of Valued Associate #679: Cesar Manara Unicorn Meta Zoo #1: Why another podcast?Success / Error message on ajax request$.widget is not a function when loading a homepage after add custom jQuery on custom themeHow can bind jQuery to current document in Magento 2 When template load by ajaxRedirect page using plugin in Magento 2Magento 2 - Update quantity and totals of cart page without page reload?Magento 2: Quote data not loaded on knockout checkoutMagento 2 : I need to change add to cart success message after adding product into cart through pluginMagento 2.2.5 How to add additional products to cart from new checkout step?Magento 2 Add error/success message with knockoutCan't validate Post Code on checkout page

                                        Fil:Tokke komm.svg

                                        Where did Arya get these scars? Unicorn Meta Zoo #1: Why another podcast? Announcing the arrival of Valued Associate #679: Cesar Manara Favourite questions and answers from the 1st quarter of 2019Why did Arya refuse to end it?Has the pronunciation of Arya Stark's name changed?Has Arya forgiven people?Why did Arya Stark lose her vision?Why can Arya still use the faces?Has the Narrow Sea become narrower?Does Arya Stark know how to make poisons outside of the House of Black and White?Why did Nymeria leave Arya?Why did Arya not kill the Lannister soldiers she encountered in the Riverlands?What is the current canonical age of Sansa, Bran and Arya Stark?